Tải bản đầy đủ (.pdf) (37 trang)

CHUẨN BỊ KIẾN THỨC ÔN THI TN THPT 2011 pdf

Bạn đang xem bản rút gọn của tài liệu. Xem và tải ngay bản đầy đủ của tài liệu tại đây (322.88 KB, 37 trang )

CHUẨN BỊ KIẾN THỨC ÔN THI TN THPT 2011
PRACTICE TEST 1
QUESTIONS 1-3: Choose one word whose underlined part is pronounced differently from the rest in
each group. Identify your answer by circling the corresponding letter A, B, C or D. Mark your choice on
the answer sheet.
1. A. resume B. statistics C. position D. designer
2. A. helps B. laughs C. likes D. arrives
3. A. filled B. missed C. ploughed D. watched
QUESTIONS 4-5: Choose one option A, B, C or D corresponding to the word whose main stress is on the
syllable in the position which is different from that of the others. Identify your choice by circling letter A,
B, C or D on the answer sheet.
4. A. teacher B. village C. police D. worker
5. A. discuss B. attract C. suppose D. visit
Questions 6 - 25: These are incomplete sentences. Choose A, B, C or D as your best choice to complete
each sentence and then identify your answer by circling letter A, B, C or D on the answer sheet.
6. The conical leaf hat is made ____________ a special kind __________ bamboo and young and soft
palm leaves.
A. in/ for B. of/ of C. on/ for D. from/ of
7. Few Asian students agreed _____________ the American students' view that wives and husbands
should share all thoughts.
A. to B. on C. for D. with
8. The wedding day was __________ chosen by the parents of the groom.
A. careless B. careful C. carelessly D. carefully
9. Peter usually helps his mother with _________ .
A. housework B. homework C. chores D. household
10. _____ __ at someone is considered as rude.
A. Looking B. Waving C. Greeting D. Pointing
11. He went to the cinema__________he had done his homework.
A. before B. until C. by the time D. after
12. _________ ten minutes earlier, you would have got a better seat.
A. Had you arrived B. If you hadn't arrived C. Were you arrived D. If you arrived


13. The conical leaf hat is one of the typical features of the Vietnamese ______.
A. education B. culture C. society D. language
14. _________ I moved in my new apartment, my neighbours have come to my house twice.
A. Because B. Since C. When D. After
15. To Americans, it is impolite to ask someone about age, ___________ and salary.
A. marry B. married C. marriage D. marrying
16. The _________way to apologize is to say 'Tm sorry."
A. most simple B. simplest C. more simple D. simpler
17. A: "Would you like to go to the cinema with me tonight?"
B: "Let me ask my parents for _________ permission."
A. a B. an C. the D. no article
18. When we came to their house, they __________ chess.
A. were playing B. are playing C. played D. have been playing
19. Since I wanted to get the bill, I tried to attract the waiter's __________ .
A. attention B. agreement C. assistance D. eyes
20. They are a close-knit family and very __________ of one another.
A. supportive B. support C. supported D. supporting
21. Compulsory education in Vietnam starts at the __________ level.
A. kindergarten B. lower secondary C. upper secondary D. primary
22. State schools are schools ____________ all children can attend without paying tuition fees.
A. which B. from which C. to which D. in which
23. Primary education is .a stage of study __________ children aged from 5 to 11 years old.
A. to B. for C. in D. with
24. The National Curriculum _________ by the government.
A. will set B. set C. is setting D. is set
25. The school year in Vietnam _________ divided into two terms.
A. was B. are C. is D. is being
QUESTIONS 26 - 33: Read the following passage carefully and then choose the best option to fit each
space. Identify your choice by circling letter A, B, C or D on the answer sheet.
Although most families in the United States enjoy a high living standard, many American women have

to work (26) ______ at home and in their offices. Despite their husbands' (27) ______ of the housework,
women have to do (28) ______ of the housekeeping tasks. In single-parent families, the mothers may not
have (29) ______ children to take care of, but they have to (30) ______ their families alone and have to
provide for all the (31) ______ money. Therefore, the (32) ______ mothers have to work very hard to earn
their living. Moreover, in spite of the fact that the single mothers have all the (33) ______ conveniences at
their homes, they find that bringing up their teenage children alone is not easy.
26. A. hardly B. hard C. fast D. equally
27. A. share B. need C. refusal D. responsibility
28. A. best B. most C. fewest D. least
29. A. few B. much C. many D. some
30. A. raise B. earn for C. keep D. nurse
31. A. homecoming B. housewarming C. household D. housekeeping
32. A. dependent B. lonely C. alone D. single
33. A. expensive B. saving C. comfortable D. modern
QUESTIONS 34 - 38: Read the following passage carefully and answer the questions that follow by
circling letter A, B, C or I) as the correct answers and then mark your choice on the answer sheet.
Like schools in Britain and other English speaking countries, those in the U.S have also always stressed
"character" or "social" skills through extracurricular activities, including organised sports. Because most
schools start at around 8 o'clock every morning and classes often do not finish until 3 or 4 o'clock in the
afternoon, such activities mean that many students do not return home until the early evening. There is
usually a very broad range of extracurricular activities available. Most schools, for instance, publish their
own student newspapers, and some have their own radio stations. Almost all have student orchestras, bands,
and choirs.
Many different sports are also available and most schools share their facilities - swimming pools, tennis
courts, tracks, and stadiums - with the public. Often the students themselves organize and support school
activities and raise money through car washes, baby-sitting, or by mowing lawns. Parents and local business
often also help a group that, for example, has a chance to go to a state music competition, to compete in
some sports championship, or take a camping trip. Such activities not only give pupils a chance to be
together outside of normal classes, they also help develop a feeling of "school spirit" among the students and
in the community.

34. How many hours do the children stay at school a day?
A. eight B. five C. six D. four
35. Which extracurricular school activities are not mentioned?
A. writing newspapers B. singing in a choir C. helping old people D. working in a radio station
36. Which sports facilities are not mentioned in the text?
A. swimming pools B. tennis courts C. golf course D. stadiums
37. What do the students do to support school activities?
A. look after a baby B. repair cars C. sell lawn movers D. sell newspapers
38. Which activities are not helped by parents and local business?
A. performing at a state music competition B. working in a TV station
C. playing in a sport competition D. having a camping trip
QUESTIONS 39 - 42: Choose one option A, B, C or D corresponding to the sentence which has the same
meaning as the original one and mark your choice on the answer sheet.
39. I often share my secrets with my sister.
A. It is my sister which I often share secrets with. B. It is my sister that I often share secrets.
C. It is my sister whose I often share secrets with. D. It is my sister with whom I often share secrets.
40. My mother is very busy at work but she takes good care of her children.
A. In spite of being busy at work, my mother takes good care of her children.
B. My mother takes good care of her children but she is very busy at work.
C. Though my mother was very busy at work my mother took good care of her children.
D. Although my mother was very busy at work she did not take good care of her children.
41. "You'd better think carefully before applying for that job.” She said to me.
A. She said me to think carefully before applying for that job.
B. She say me to think carefully before applying for that job.
C. She advise me to think carefully before applying for that job.
D. She advised me to think carefully before applying for that job.
42. Anne started studying non-verbal forms of communication three years ago.
A. Anne has studying non-verbal forms of communication for three years.
B. Anne has study non-verbal forms of communication for three years.
C. Anne has been studying non-verbal forms of communication for three years.

D. Anne has been studying non-verbal forms of communication since three years.
QUESTIONS 43 - 50: Each of the following sentences has four underlined words or phrases marked A,
B, C, and D. Choose the word or phrase which must be changed in order for the sentence to be correct by
circling letter A, B, C or D and then mark your choice on the answer sheet.
43. The British (A) are generally less will (B) to talk about (C) personal matters (D).
44. Many people (A) may work (B) long hours and have (C) little time for an (D) social life.
45. Some people (A) become close friends (B) with people he (C) work with (D).
46. They (A) have translated Nguyen Du's novel (B) "Kieu" into (C) several language (D).
47. They sold (A) computers commercial (B) for the first time (C) in the 1950s (D).
48. People (A) use computer (B) to forecast the (C) weather or to control robots (D).
49. They could (A) publish his new (B) English - Vietnamese dictionary (C) next month (D).
50. A new school (A) for disabled children (B) will be build (C) here next month (D).

THE END
PRACTICE TEST 2
QUESTIONS 1-2: Choose one word whose underlined part is pronounced differently from the rest in
each group. Identify your answer by circling the corresponding letter A, B, C or D. Mark your choice on
the answer sheet.
1. A. hands B. occasions C. associates D. others
2. A. frightened B. stamped C. walked D. laughed
QUESTIONS 3-5: Choose one option A, B, C or D corresponding to the word whose main stress is on the
syllable in the position which is different from that of the others. Identify your choice by circling letter A,
B, C or D on the answer sheet.
3. A. compulsory B. curriculum C. nursery D. certificate
4. A. average B. aquatic C. athletic D. available
5. A. mathematics B. sociology C. chemistry D. archaeology
QUESTIONS 6 - 20: These are incomplete sentences. Choose A, B, C or D as your best choice to
complete each sentence and then identify your answer by circling letter A, B, C or D on the answer sheet.
6. ______ should try to make good impression on interviewers.
A. Apply B. Applicants C. Applying D. Applied

7. In England public schools are also called _____________schools.
A. private B. state C. free paying D. independent
8. Our company wants to take in those who are _____________for the job.
A. available B. useful C. qualified D. good
9. Universities send letters of_____________ to successful candidates by post.
A. accept B. acceptably C. acceptable D. acceptance
10. In your …………. letters you should mention your education, qualifications and work experience.
A. applicant B. apply C. application D. applicable
11. Don't forget to say goodbye to the interviewer_________ leaving the office.
A. before B. after C. while D. when
12. Children stay in secondary school _________ the age of sixteen.
A. for B. until C. at D. in
13. This clerk will provide us with the information about applying___________ universities in the UK.
A. at B. to C. for D. with
14. During the interview you should concentrate on__________ the interviewer is saying.
A. when B. that C. what D. where
15. Mary searched all over the house __________she could not find the key.
A. but B. although C. however D. because
16. Does anyone know _________ this hat might belong?
A. who B. whom C. whose D. to whom
17. If it rained heavily, the fields _________ flooded now.
A. will be B. had been C. would be D. can be
18. A _________is a person who takes people to places of interest.
A. writer B. pilot C. tourist guide D journalist
19. The national curriculum is made _______________ of the subjects such as: English, Maths, Chemistry
and so on.
A. in B. up C. on D. from
20. ______ wants to work for this company can send their application letter by post.
A. Who B. That C. Whoever D. Whom
QUESTIONS 21 - 24: Read the following passage and complete the sentences that follow by circling the

corresponding letter A, B, C or D. Mark your choice on the answer sheet.
Susan was very nervous about her interview. For at least three weeks before it she was worried about it.
She really wanted the job but she knew that a lot of people wanted to do that too. She had been told that
there were a great many applicants for it, so she prepared herself. She made notes of what she might be
asked and of what she wanted to ask. When the day came, she arrived half an hour early. There were six
other people waiting to be interviewed. They looked much more confident than her. She began to feel even
more nervous. One by one, the others were called. Each of them came out looking satisfied. Susan was the
last one to be called into the interview room. She had decided by then that she had no chance of getting the
job, so she felt relaxed as she walked in; she felt that she had nothing to lose. The three interviewers were all
very serious and they didn't seem to be interested in her. She forgot all the answers she had prepared and said
the first things that came into her head. Afterwards she was sure she wouldn't get the job, but two days later
she got a letter telling her she had been chosen because she had been the only one who acted naturally.
21. Susan was told that there were many ______ for the job.
A. application B. applicant C. apply D. applicants
22. There were only ______ interviewees.
A.6 B.7 C.8 D. 9
23. Susan felt even more nervous than before because the other people .
A. acted better than her B. looked more confident than her
C. looked more relaxed than her D. knew that she had no chance of getting the job
24. Susan won the job because she was the only one who ____________ .
A. answered the questions naturally B. asked the questions naturally
C. acted naturally D. told naturally
QUESTIONS 25 - 33: Read the following passage carefully and then choose the best option to fit each
space. Identify your choice by circling letter A, B, C or D, Mark your choice on the answer sheet.
21, Jalan Sultan
Johore Bahru
The advertiser,
P.O. Box 345,
Kuala Lumpur,
Dear Sir,

In response to your advertisement in the "Malaysia Times" of 1st January for an office clerk. I am (25)
______ this application for your consideration. I am 20 years of (26) ______ and have passed the GCE "A "
Level with certificate in English, Mathematics and General Science. In (27) ______ have taken a Computer
(28) ______. I can type 40 words a(n) (29) ______ and have also a working knowledge of book-keeping and
accounts. I am proficient in National Language as well as English.
I am in good health and take interest (30) ______ sport. I am enclosing copies of certificate (31) ______
from my school. If you grant me an interview, I shall be happy to come to your office any time convenient
(32) ______ you.
I am looking forward to (33) ______ from you soon.
Yours faithfully,
25. A. making B. writing C. submitting D. sending
26. A. age B. old C. life D. lives
27. A. short B. brief C. opinion D. addition
28. A. course B. lesson C. material D. subject
29. A. minute B. hour C. second D. day
30. A. on B. at C. in D. about
31. A. obtaining B. studying C. having D. obtained
32. A. to B. for C. with D. on
33. A. hearing B. hear C. hears D. heard
QUESTIONS 34 - 40: Choose one option A, B, C or D corresponding to the sentence which has the same
meaning as the original one and mark your choice on the answer sheet.
34. Hanoi is famous for its beauty.
A. It is the beauty of Hanoi that makes it famous. B. Hanoi is a beautiful city.
C. It is Hanoi whose beauty attracts many foreign visitors. D. Because Hanoi is famous, it is beautiful.
35. Money must be easy to carry about.
A. Money must be lasting. B. Money must be easy to recognize.
C. Money must be easy to bring along. D. Money must be used easily.
36. It is difficult to start looking for a job at my age.
A. I'm young, so it is difficult to start looking for a job. B. It is difficult for me to get employed at my age.
C. Getting employed at such ages is also difficult. D. I don't think I can start looking for a job now.

37. The USA is a country of high youth unemployment.
A. The USA is a country of young people. B. It is the USA that has a great number of young people.
C. High youth unemployment is found in the USA. D. We find high youth unemployment a problem in the USA.
38. Life in Vietnam in the 2500 will be very different.
A. Vietnam will be very different in the 2500. B. The 2500 will see great changes in Vietnam.
C. The 2500 will see great differences in life in Vietnam.
D. People of Vietnam will make great changes for the country.
39. Sue lives in a house. The house is opposite my house.
A. Sue lives in the house which is opposite my house. B. Sue lives in the house who is opposite my house.
C. Sue lives in the house where is opposite my house. D. Sue lives in the house and which is opposite my house.
40. The plants may develop differently. The plants live on that island.
A. The plants which live on that island may develop differently.
B. The plants lives on that island may develop differently.
C. The plants which lives on that island may develop differently.
D. The plants which live that island may develop differently.
QUESTIONS 41 - 50: Each of the following sentences has four underlined words or phrases marked A,
B, C, and D. Choose the word or phrase which must be changed in order for the sentence to be correct by
circling letter A, B, C or D and then mark your choice on the answer sheet.
41. What (A) have they say (B) about their (C) marriages (D)?
42. You should to take (A) these tablets (B) and rest for (C) a couple of days (D).
43. Bacteria is (A) helpful (B) to life (C) on earth (D).
44.I will ask (A) my students (B) to repeat the (C) sentences again (D).
45. I'm afraid (A) I can't (B) come with (C) you on tomorrow (D).
46. We usually (A) have English lesson (B) twice (C) a week (D).
47. They never have (A) breakfast (B) before (C) having bath (D).
48. When he came (A), we were (B) having (C) dinners (D).
49. She likes (A) her job but (B) does not (C) like wear uniforms (D).
50. Children can't (A) start school (B) not until (C) they are six (D).
PRACTICE TEST 3
QUESTIONS 1-3: Choose one word whose underlined part is pronounced differently from the rest in

each group. Identify your answer by circling the corresponding letter A, B, C or D. Mark your choice on
the answer sheet.
1. A. science B. client C. review D. variety
2. A. prepare B. care C. share D. cage
3. A. pay B. clay C. essay D. said
QUESTIONS 4-5: Choose one option A, B, C or D corresponding to the word whose main stress is on the
syllable in the position which is different from that of the others. Identify your choice by circling letter A,
B, C or D on the answer sheet.
4. A. design B. essay C. caring D. special
5. A. escape B. marry C. prepare D. recount
Questions 6 - 30: These are incomplete sentences. Choose A, B, C or D as your best choice to complete
each sentence and then identify your answer by circling letter A, B, C or D on the answer sheet.
6. In the past the earth was ______ to be flat.
A. created B. evolved C believed D. built
7. We'll talk about those ______ problems are related to psychology.
A. what B. which C. whose D. who
8. In my opinion Anderson's reputation as an artist is very ______.
A. overrated B. overpowered C. overstated D. overlooked
9. The city is determined to rid itself of problems with drainage, waste, air and noise pollution ______ to
make Vietnam's aged capital a clean and green city.
A. so that B. in order that C. to order D. so as
10. The city administration plans to invest VND 26 billion to ______ polluted water.
A. solve B. treating C. solving D. treat
11. You will find John ______ difficult to understand at first, as he has a slight impediment in his speech.
A. rather B. hardly C. over D. enough
12. They searched ______ for the missing documents but they were never found.
A. to and fro B. high and low C. up and down D. back to back
13. If one ______ a killing, one must expect to suffer the consequences.
A. creates B. makes C. commits D. shows
14. The habit of taking cold baths in winter is said to be ______ to the English.

A. individual B. personal C. intelligent D. peculiar
15. You'd better not argue with them, ______ you'll be fined.
A. if B. in case C. unless D. otherwise
16. ______ from automobiles causes a large percentage of all air pollution.
A. Exhaust B. Pollution C. Gas D. Air
17. Those ______ showed signs of psychological effects behaved very differently.
A. how B. who C. where D. whose
18. Heavily polluted plants will be closed down only if the matter is ______ of by the Prime Minister.
A. disapproved B. disapproving C. approved D. approving
19. Flood ______ from the heavy rain destroyed the crops in the region.
A. resulted B. to result C. results D. resulting
20. Increasing industrialization is challenging the province to ______ every effort to control pollution.
A. do B. create C. make D. give
21. ______ are still living, others are dead.
A. Some types of germs B. While some types of germs
C. There are some types of germs D. But some types of germs
22. You won't be allowed to teach here ______ you show your qualification.
A. since B. unless C. for D. while
23. Did the man say he ______ already sent the application form?
A. should B. would C. have D. had
24. In the United States, ______ is generally the responsibility of municipal government.
A. for water treatment B. water treatment C. where water treatment D. in which water treatment
25. It would be hard to process the data ______ using a computer.
A. for B. with C. since D. without
26. "This room is filled with smoke". "Yes, this is a ______ room."
A. smoke-filled B. smoke-filling C. filling-smoke D. smoking-filled
27. We'll talk about the questions ______ logical thought.
A. require B. requiring C. required D. are requiring
28. You have learned many things useful there, ______ you?
A. have B. do C. haven't D. don't

29. Most folk songs are ballads ______ have simple words and tell simple stories.
A. what B. although C. when D. that
30. My brother got ______ with doing the same thing for days in his office.
A. boring B. bored C. boringly D. boredom
QUESTIONS 31 - 35: Read the following passage carefully and then choose the best option to fit each
space. Identify your choice by circling letter A, B, C or D on the answer sheet.
Most people work to earn a living, and produce goods and services. Goods are (31) ______ agricultural
(like maize and milk) or manufactured (like cars and paper). Services are such things (32) ______ education,
medicine, and commerce. Some people provide products. An example can be a man who produces goods
such as rice. Some provide services. In a hotel, services can range from cleaning, washing to arranging tours
within and outside the city where (33) ______. Other people provide both goods and services. For example,
in the same garage a man may buy a car or some service which helps him to maintain his car.
The work people do is called economic activity. All economic activities together make up the economic
system of a town, a city, a country, or the world. Such an economic system is the sum (34) ______ and what
they want. The work people undertake provides either what they need or provides the money with (35)
______ they can buy essential commodities. Of course, most people hope to earn enough money to buy
commodities and services which are non-essential but which provide some particular personal satisfaction,
like toys for children, visits to the cinema, and books.
31. A. either B. both C. neither D. only
32. A. else B. for C. with D. as
33. A. are the guests staying B. the guests are staying C. are staying the guests D. staying the guests are
34. A. what people do B. of total what people do
C. total of what people do D. of what people do in total
35. A. that B. without which C. the money D. which
QUESTIONS 36 - 40: Read the following passage carefully and answer the questions that follow by
circling letter A, B, C or D as the correct answers and then mark your choice on the answer sheet.
A greenhouse is a building whose sides and roof are made of glass so that the temperature inside is
magnified. And it's used to grow plants that need high temperatures. This is to illustrate an example of how
man could be causing changes to the climate. These changes result from increasing the carbon dioxide levels
in the atmosphere, thus raising the surface temperature of the Earth. And this is known as the "Greenhouse

Effect."
C02 is a normal component of the atmosphere and until recently has not been considered an air
pollutant. But average global C02 concentrations have been increasing since 1860, with particularly sharp
increase in 1958. The main reason for this continuous increase in C02 build-up is the burning of fossil fuels.
During the past 100 years, the C02 content of the atmosphere has already risen by about 15% - from about
290 to about 340 parts per million. And it's still rising.
Although the C02 content of the atmosphere is only about 0.032%, it's a major factor in determining
average global temperature. Incoming sunlight consists of many wavelengths including some very dangerous
ones. But ozone in the upper atmosphere and water vapor and C02 in the lower atmosphere filter out or
destroy most of the harmful wavelengths, so what reaches the Earth is mostly visible light. It's absorbed by
land, sea and cloud and is reradiated in the atmosphere as longer wavelength infrared radiation, or heat, as
the Earth cools. Much of this infrared radiation is absorbed by C02. The C02 then radiates a portion of the
absorbed heat energy back to the Earth to warm the atmosphere. Rather like the glass in a greenhouse or a
car window on sunny day, C02 in the atmosphere acts as a one-way filter that allows visible light to enter the
Earth's atmosphere, but prevents longer wavelength heat radiation from leaving. Assuming that energy is
arriving from the sun at a constant rate, then as the level of C02 increases, the average surface temperature of
the earth should rise.
36. What is the purpose of using a greenhouse?
A. to give an example of atmospheric changes caused by man
B. to magnify temperature to grow plants requiring high temperatures
C. to give an example of buildings whose sides and roof are made of glass
D. to indicate the rising level of C02 in the earth's atmosphere
37. What is mainly discussed in paragraph 1?
A. The sides and roof of a greenhouse are made of glass
B. Plants need high temperatures to grow
C What is the "Greenhouse Effect"?'
D. A greenhouse is used for growing plants
38. Why is C02 considered an air pollutant?
A. C02 is a normal component of the atmosphere
B. Global C02 concentrations have been increasing

C. C02 is a by-product of the burning of fossil fuels
D. C02 raises the surface temperature of the Earth
39. What is not true according to the information from the passage?
A. The level of C02 has risen by as much as 15%
B. The sun emits some very dangerous wavelengths
C. A portion of sunlight is filtered out before entering the Earth
D. C02 allows sunlight to enter and leave the Earth's atmosphere
40. What is the best title for the passage?
A. The "Greenhouse Effect" and how C02 warms the atmosphere
B. The level of C02 at present compared with that in the past
C. Ozone and water vapor with filtering out harmful wavelengths
D. The "Greenhouse Effect" used for growing plants
QUESTIONS 41 - 45: Choose one option A, B, C or D corresponding to the sentence which has the same
meaning as the original one and mark your choice on the answer sheet.
41. These convicts were sent to Australia as punishment for their crimes.
A. These criminals were punished and sent to Australia.
B. These convicts punished themselves for their crimes by going to Australia.
C. They sent these convicts to Australia, then punished them.
D. Because of their crimes, these convicts were sent to Australia as punishment.
42. If students have problems, you can ask questions to check their understanding.
A. I advice you to ask questions to check their understanding.
B. It's better to check their understanding by asking questions.
C. Questions can be asked to check their understanding if students have problems.
D. Asking questions to check their understanding if they have problems.
43. / don't think Jerry will win the tennis match.
A. In my opinion, Jerry will not likely to win the tennis match.
B. I will never think Jerry will win the tennis match.
C. The fact is that Jerry won't win the tennis match.
D. Jerry will win the tennis match, I don't think that.
44. After days of searching, the captain found a fine harbor.

A. The captain found a fine harbor; then he searched for days.
B. The captain had searched for days before he could find a fine harbor.
C. It took the captain some time to search for a fine harbor.
D. The captain spent many days on exploring the fine harbor in the new place.
45. There are many factors contributing to air pollution.
A. Air pollution results in many consequences.
B. Air pollution is the result of burning forests.
C. Factors contributing to air pollution are numerous.
D. Air pollution is contributing to these phenomena.
Chuẩn bò kiến thức ôn thi tốt nghiệp THPT và tuyển sinh ĐH, CĐ năm 2008-2009
Nguyen Thanh Tn – GV Tiêng Anh – THPT T_ Ky - T_ Ky - H_i Dư_ng Page 9 of 40
QUESTIONS 46 - 50: Each of the following sentences has four underlined words or phrases marked A,
B, C, and D. Choose the word or phrase which must be changed in order for the sentence to be correct by
circling letter A, B, C or D and then mark your choice on the answer sheet.
46. The aim of this exercises (A) is to practise (B) using will for (C) a future fact or prediction (D).
47. Heavily (A) polluted factories will be closed (B) down sooner (C) or latest (D).
48. Tell students (A) are going (B) to hear three short (C) conversations about food (D).
49. I will (A) never forget (B) what has he (C) done to me (D).
50. Sue enjoyed the (A) film is so much (B) that she said she would (C) go to the movie to (D) see it again.
PRACTICE TEST 4
QUESTIONS 1-3: Choose one word whose underlined part is pronounced differently from the rest in
each group. Identify your answer by circling the corresponding letter A, B, C or D. Mark your choice on
the answer sheet.
1. A. cheerful B. charity C. machine D. chance
2. A. field B. belief C. brief D. friend
3. A. between B. cheese C. cheep D. cheer
QUESTIONS 4-5: Choose one option A, B, C or D corresponding to the word whose main stress is on the
syllable in the position which is different from that of the others. Identify your choice by circling letter A,
B, C or D on the answer sheet.
4. A. between B. cherish C. chicken D. kitchen

5. A. exciting B. charity C. convenient D. surprising
QUESTIONS 6 - 30: These are incomplete sentences. Choose A, B, C or D as your best choice to
complete each sentence and then identify your answer by circling letter A, B, C or D on the answer sheet.
6. Never in my life ______ such a beautiful sunset.
A. I have seen B. I do see C. have I seen D. do I see
7. I will phone you ______ I get there.
A. as well as B. as much as C. as soon as D. as early as
8. If we heat water to 100 degrees Celsius, it ______.
A. would boil B. will be boiling C. is boiling D. boils
9. After finishing the presentation, I suggested ______ to the cinema.
A. us to go B. that we to go C. going D. we going
10. If you do not learn seriously, ______ to understand the subject well.
A. you will never be able B. will you never be able
C. never you will be able D. will never you be able
11. Early carpenters, having ______ nails, had to use wooden pegs to secure their constructions.
A. not B. neither C. no D. none
12. Not ______ 1975 did we liberate the South.
A. since B. for C. until D. as
13. People who reverse the letters of words ______ to read suffer from dyslexia.
A. when trying B. if they tried C. when tried D. if they try
14. Featured at the Henry Ford Museum ______ of antique cars dating from 1865.
A. is an exhibit B. an exhibit C. are an exhibit D. an exhibit is
15. The purpose of a labor union is to improve the working conditions and ______ its members.
A. protecting B. to protect C. in protection D. for protection
16. We cannot imagine ______ life would be like without music.
A. what B. when C. where D. how
17. I do believe that his study there will be ______ great benefit to his teaching career later.
A. on B. to C. in D. of
8. The formal statement of this process is ______ the principle of natural selection.
A. referred B. called C. known D. regarded

19. What ______ is protecting the area and these endangered species.
A. in environmentalists are interested B. are environmentalists interested in
C. are interested in environmentalists D. environmentalists are interested in
20. "What's wrong with you today? Did you get out of bed on the wrong ______ ?"
A. end B. edge C. foot D. side
Chuẩn bò kiến thức ôn thi tốt nghiệp THPT và tuyển sinh ĐH, CĐ năm 2008-2009
Nguyen Thanh Tn – GV Tiêng Anh – THPT T_ Ky - T_ Ky - H_i Dư_ng Page 10 of 40
21. From these and ______ observations, Darwin put together a concept of natural selection.
A. another B. other C. others D. other's
22. A serious study of physics is impossible without ______ knowledge of mathematics.
A. several B. amount of C. some D. few
23. Hardly ______ play truant. Something must have happened to them.
A. do they B. they do C. have they D. they have
24. It's very cold in here, do you mind ______ I close the window?
A. whether B. as if C. if D. for
25. If you had asked him, he helped you to solve the problem.
A. will have B. ought to have C. should have D. would have
26. Sidney always gets high marks because her compositions are ______.
A. well extremely written B. extremely well written
C. written well extremely D. well written extremely
27. "Whose book is it?" - "I don't know, but I think ______ ."
A. it is Jim B. it is Jim's C. it is belonging to Jim D. it belongs to Jim's
28. We ______ for an hour when the bus finally came.
A. waited B. have waited C. have been waiting D. had been waiting
29. ______ Mark, is attending the lecture now.
A. Right now B. At that time C. Because of the time D. My friend
30. The tetracyclines, ______ antibiotics, are used to treat infections.
A. are a family of B. being a family C. a family of D. their family is
QUESTIONS 31 - 35: Read the following passage carefully and then choose the best option to fit each
space. Identify your choice by circling letter A, B, C or D on the answer sheet.

A friend of mine has just come back from her holiday in Australia. She's telling us about her holiday
there. Here is her description of this fantastic place. "We arrived at Sydney airport really early in the
morning. The view from the (31) ______ Sydney harbor was fantastic. We were really exhausted but we
wanted to have (32) ______ around Sydney, so we went sightseeing straight away. We went to the Opera
House First. It's even more beautiful than in the photos - really (33) ______. We then went to the Rocks,
which is the old section of Sydney and which is now very pretty, particularly at night. The next morning we
went on Harbor Cruise. On the following day we flew up to Cairns. The scenery was wonderful there. We
especially liked the Barrier Reef (34)______ colorful fish and clear blue water. After that, we decided to go
to (35) ______ quiet. We flew to Seaford. We didn't see many tourists there. However, I love this place.
There's so much wildlife - a bird-watcher's dream!"
31. A. looking down over plane B. plane looking down over
C. plane down over looking D. over plane looking down
32. A. a look a bit of B. a bit look of a C. a bit of a look D. of a bit a look
33. A. spectator B. spectate C. spectacular D. spectacularly
34. A. with its B. for its C. together its D. like its
35. A. nowhere B. wherever C. everywhere D. somewhere
QUESTIONS 36 - 40: Read the following passage carefully and answer the questions that follow by
circling letter A, B, C or D as the correct answers and then mark your choice on the answer sheet.
Why are there so many grandmothers and so few grandfathers? In other words, why do men die younger
than women? Is it because men are afraid of getting old and helpless and so they prefer to die before that
happens? Perhaps they fear to be left alone by their women and so decide to do the leaving first. Many
explanations are given for the fact that men die earlier than women. Men are stronger physically, yet women
can hang on longer to life. Both men and women are emotional creatures but women are not afraid or
ashamed to cry while men refuse to do so. They are afraid of being thought "soft". Some men, when they are
upset, play loud music or dig in the garden to relieve their feelings. Many men like an orderly life so that
they bury themselves in their work or want a wife with some children or long to take part in wars where
there is a definite chain of command. There is a leader to give orders and a known enemy to fight against
and defeat. All these make men feel they live in a rational world.
So the majority of men like a rational world. Rationality is fine but it does not include everything that
makes life joyful and fun or even messy and frustrating. When a man refuses to cry he is refusing to accept

that his emotions are part of him. Of course some men do not follow this pattern. Bob Hawke is capable of
crying in public whereas Margaret Thatcher is probably incapable of crying at any time. We need to cry
Chuẩn bò kiến thức ôn thi tốt nghiệp THPT và tuyển sinh ĐH, CĐ năm 2008-2009
Nguyen Thanh Tn – GV Tiêng Anh – THPT T_ Ky - T_ Ky - H_i Dư_ng Page 11 of 40
because that shows our ability to suffer. If we do not suffer we are not really alive at all. Suffering can be
creative or destructive. If we can all learn to cry and laugh and shout and dance openly, we are living
creatively and adding something to the human race. We often say men suffer more from stress and therefore
they die earlier. Are we not saying in another way that they do not know how to suffer in the right way -
with tears and laughter instead of silence - and so they are miserable and just give up? That is just my theory
of course. Have you a better one?
36. What does the word "that" (the underlined word) in the passage refer to?
A. The fact that men die younger than women
B. The fact that they fear to be left alone
C. The fact that men get old and helpless
D. The fact that they decide to do the leaving first
37. What is found the same for both men and women?
A. They like to cry B. They are physically strong
C. They want to play loud music D. They are emotional creatures
38. Which paragraph explains why men die earlier than women?
A. Paragraph 1 B. Paragraph 2 C. Paragraph 3 D. Paragraph 4
39. What's the writer's opinion toward crying?
A. It is a good thing for people to cry B. To be silent is better than to cry
C. It is all right for women to cry but shameful for men D. Tears are not real signs of suffering
40. What's the main question the article is intended to discuss?
A. Why men like a rational world B. Why women are more emotional than men
C. Why suffering is different in men and women D. Why men die earlier than women
QUESTIONS 41 - 45: Choose one option A, B, C or D corresponding to the sentence which has the same
meaning as the original one and mark your choice on the answer sheet.
41. Most information was collected by direct observations of its behaviours.
A. Many direct observations have been made in its behaviours.

B. Almost the information was the result of direct observations of its behaviours.
C. Direct observations of its behaviours made for collecting most information.
D. Almost the information came from its behaviours for direct observations.
42. They try to prevent the reader from vocalizing.
A. They try to stop the reader from vocalizing.
B. The reader tries to prevent vocalizing.
C. The reader tries to prevent them from vocalizing.
D. They try to prevent vocalizing from the reader.
43. This passage focuses on helping students read faster.
A. How to read this passage faster and faster.
B. Reading faster is the content of this passage.
C. The purpose of this passage is to help students read more quickly.
D. The purpose of this passage is to helping students read more quickly.
44. All of the students, but Peter went to the meeting.
A. Peter went to the meeting; and so did all of the students.
B. Except for Peter, all of the students went to the meeting.
C. All of the students, especially Peter went to the meeting.
D. All of the students, including Peter went to the meeting.
45. We take great pride in offering the best service in town.
A. Our service is the best. That's why people can see our pride.
B. We are very proud of offering the best service in town.
C. The best service in town is offered to people with great pride.
D. No-one takes great pride in offering the best service in town, but we do.
QUESTIONS 46 - 50: Each of the following sentences has four underlined words or phrases marked A,
B, C, and D. Choose the word or phrase which must be changed in order for the sentence to be correct by
circling letter A, B, C or D and then mark your choice on the answer sheet.
46. The jacket with (A) three differently (B) colors belongs to (C) Hai, my brother (D).
47. I often (A) spend some money each month (B), about $50, to buy (C) books on (D) economics.
48. She must have (A) her reasons (B) where (C) she doesn't want to (D) tell.
Chuẩn bò kiến thức ôn thi tốt nghiệp THPT và tuyển sinh ĐH, CĐ năm 2008-2009

Nguyen Thanh Tn – GV Tiêng Anh – THPT T_ Ky - T_ Ky - H_i Dư_ng Page 12 of 40
49. Production (A) is the (B) process growing (C) or making food, goods, or (D) materials.
50. As you (A) can see, costs of living have been (B) reduced for (C) 20% over the past (D) year.
PRACTICE TEST 5
QUESTIONS 1-2: Choose one option A, B, C or D corresponding to the word whose main stress is on the
syllable in the position which is different from that of the others. Identify your choice by circling letter A,
B, C or D on the answer sheet.
1. A. penalty B. wildness C. reunite D. fascinating
2. A. unnoticed B. composed C. precision D. telescope
QUESTIONS 3-5: Choose one word whose underlined part is pronounced differently from the rest in
each group. Identify your answer by circling the corresponding letter A, B, C or D. Mark your choice on
the answer sheet.
3. A. exactly B. exert C. exam D. excellent
4. A. invite B. shrine C. village D. primary
5. A. depends B. years C. temples D. parents
QUESTIONS 6 - 25: These are incomplete sentences. Choose A, B, C or I) as your best choice to
complete each sentence and then identify your answer by circling letter A, B, C or D on the answer
sheet.
6. Helen's parents were very pleased when they read her school ______.
A. report B. papers C. diploma D. account
7. Martin has quite a good ______ of physics.
A. capacity B. pass C. understanding D. head
8. In Britain, children start ______ school at the age of five.
A. primary B. secondary C. nursery D. kindergarten
9. My favourite ______ at school was history.
A. topic B. class C. theme D. subject
10. It's time for break. The bell has ______.
A. gone B. struck C. rung D. sounded
11. Neither the students nor their lecturer ______ English in the classroom.
A. uses B. use C. are using D. have used

12. Some people say that Sue is unfriendly, but she is always very nice ______ me.
A. to B. of C. at D. on
13. They ______ or more than five miles but they did not stop to rest.
A. were walking B. have walk C. had been walking D. have been walking
14. Her relatives didn't do anything to help her, and her friends ______.
A. didn't neither B. didn't too C. didn't either D. did too
15. People who ______ crimes of violence should be punished.
A. make B. create C. commit D. cause
16. We can ______ not only through words but also through body language.
A. talk B. transfer C. interpret D. communicate
17. ______ and interest are the two major factors that make you succeed in your work.
A. Friendliness B. Friendly C. Friendship D. Unfriendly
18. If John ______ an alarm, the thieves wouldn't have broken into his house.
A. to install B. had installed C. have installed D. was installed
19. The 22nd SEA Games were well prepared and successfully ______ although it as the first time Vietnam
hosted.
A. organization B organize C. organized D. organizational
20. The spirit of the Asian Games has been ______ over a long period of time, and passed from one Games
to the next spreading messages of unity, warmth, and friendship.
A. gone over B. put off C looked for D. built up
21. Eating and living in this country is becoming ______ expensive.
A. more and more B. too C. less D. so
22. He kept his job ______ the manager had threatened to sack him.
A. despite B. unless C. even D. although
23. Many people used to think that women's ______ are childbearing and home taking.
Chuẩn bò kiến thức ôn thi tốt nghiệp THPT và tuyển sinh ĐH, CĐ năm 2008-2009
Nguyen Thanh Tn – GV Tiêng Anh – THPT T_ Ky - T_ Ky - H_i Dư_ng Page 13 of 40
A. abilities B. works C. roles D. task
24. Hoa: "I think women should not go to work." Hai: I ______.
A. quite agree B. a little agreed C. so agree D. rather agreed

25. After a terrible argument with his boss, she handed in her ______.
A. reservation B. reputation C. resignation D. responsibility
QUESTIONS 26 - 30: Read the following passage carefully and answer the questions that follow by
circling letter A, B, C or D as the correct answers and then mark your choice on the answer sheet.
"Where is the university?" is a question that many visitors to Cambridge ask, but no one can give them a
clear answer, for there is no wall to be found around the university. The university is the city. You can find
the classroom buildings, libraries, museums and offices of the university all over the city. And most of its
members are the students and teachers or professors of the thirty-one colleges. Cambridge was already
developing town long before the first students and teachers arrived 800 years ago. It grew up by the river
Granta, as the Cam was once called. A bridge was built over the river as early as 875.
In the fourteen and fifteen centuries more and more land was used for college buildings. The town grew
much faster in the nineteen century after the opening of the railway in 1845. Cambridge became a city in
1951 and now it has the population of over 100,000. Many young students want to study at Cambridge.
Thousands of people from all over the world come to visit the university town. It has become a famous place
all round the world.
26. Why do most visitors come to Cambridge?
A. To see the university B. To study in the colleges in Cambridge
C. To find the classroom buildings D. To use the libraries of the university
27. Around what time did the university begin to appear?
A. In the 8th century B. In the 13th century C. In the 9th century D. In the 15th century
28. Why did people name Cambridge the "City of Cambridge"?
A. Because the river was very well-known. B. Because there is a bridge over the Cam.
C. Because it was a developing town. D. Because there is a river named Granta.
29. After which year did the town really begin developing?
A.800 B. 875 C.1845 D. 1945
30. From what we read, we know that Cambridge is now ______________ .
A. visited by international tourists B. a city without wall
C. a city of growing population D. a city that may have a wall around it
QUESTIONS 31 - 40: Read the following passage carefully and then choose the best option to fit each
space. Identify your choice by circling letter A, B, C or D on the answer sheet.

There is no one who has not heard of Walt Disney: He is without doubt one of the most famous singers
in the twentieth century and (31) ______ most people know hardly anything about him.
(32) ______ he became one of the most successful men in history, he left school at the age of sixteen
and then studied art for a short time. By the (33) ______ years of this century, he had already started to
produce cartoons in Hollywood in (34) ______ with his brother Roy, who, for some reasons, never (35)
______ to become as famous as Walt Disney is perhaps most well known on account of his lovable
cartoons character. Mickey Mouse, who first (36) ______ in 1928 in a film called "Steamboat Willie". One
of the most (37) ______ cartoon films of all time is "Snow White and the Seven Dwarfs", which when it was
released in 1937, was the first full-length cartoon in the history of the cinema.
(38) ______ the 1950s, Walt Disney had became one of the world's major producers of films for
cinema and television. As Disney Productions (39) ______, its founder retained complete artistic control of
the films and he also (40) ______ on to publish books for children and cartoon strips in newspaper, featuring
such characters as Donald Duck and the Pluto Dog.
31. A. yet B. then C. already D. however
32. A. Despite B. Inspire of C. Although D. Even
33. A. primary B. early C. beginning D. initial
34. A. friendship B. partnership C. relation D. membership
35. A. achieved B. reached C. succeeded D. managed
36. A. appeared B. performed C. started D. began
37. A. common B. excited C. popular D. known
38. A. Until B. Since C. By D. To
39. A. grew B. enlarged C. increased D. succeeded
Chuẩn bò kiến thức ôn thi tốt nghiệp THPT và tuyển sinh ĐH, CĐ năm 2008-2009
Nguyen Thanh Tn – GV Tiêng Anh – THPT T_ Ky - T_ Ky - H_i Dư_ng Page 14 of 40
40. A. took B. put C pushed D. went
QUESTIONS 41 - 50: Each of the following sentences has four underlined words or phrases marked A,
B, C, and D. Choose the word or phrase which must be changed in order for the sentence to be correct by
circling letter A, B, C or D and then mark your choice on the answer sheet.
41. The primary causes (A) of species extinction or endangerment are (B) habitat destruction, commerce (C)
exploitation and pollution (D).

42. Different (A) conservation efforts have been made (B) in order to saving (C) endangered species (D).
43. Do not start (A) a book unless (B) you can see from the first little (C) pages that it is one you can easily
read and understand (D).
44. Marilyn Monroe, who was (A) a famous (B) actress, was died (C) of drug overdose (D).
45. When he was (A) at school, Anderson did not (B) show any talent in writing (C) but late (D) he became
a famous novel writer.
46. Scientists says (A) that something (B) very serious (C) is happening to (D) the Earth.
47. There will (A) be major changes (B) in climate (C) during next (D) century.
48. World temperatures (A) could increased (B) two degrees centigrade (C) by the year (D) 2040.
49. What would (A) happen if (B) the temperature (C) is warmer (D)?
50. Deforestation is occurring (A) most rapid (B) in tropical (C) regions of the world (D).
PRACTICE TEST 6
QUESTIONS 1-2: Choose one word whose underlined part is pronounced differently from the rest in
each group. Identify your answer by circling the corresponding letter A, B, C or D. Mark your choice on
the answer sheet.
1. A. hear B. fear C. pear D. ear
2. A. smiled B. worked C. picked D. jumped
QUESTIONS 3-5: Choose one option A, B, C or D corresponding to the word whose main stress is on the
syllable in the position which is different from that of the others. Identify your choice by circling letter A,
B, C or D on the answer sheet.
3. A. daughter B. decide C. provide D. enjoy
4. A. physicist B. president C. inventor D. gardener
5. A. factory B. farmer C. fairy D. farewell
QUESTIONS 6 - 24: These are incomplete sentences. Choose A, B, C or D as your best choice to
complete each sentence and then identify your answer by circling letter A, B, C or D on the answer sheet.
6. Elephants scratch themselves with sticks ______.
A. holding in their trunks B. in their trunks holding C. hold in their trunks D. held in their trunks
7. Jane suggested ______ a car.
A. me to buy B. that I buy C. I will buy D. me buy
8. The windows are dirty. They need ______.

A. be wash B. to wash C. being washed D. washing
9. I'd rather you ______ anyone what I said.
A. don't tell B. won't tell C. didn't tell D. not to tell
10. Not many people live in the north of the country. Most ______ live in the south.
A. of peoples B. persons C. population D. of the population
11. Vermont, commonly known as the Green Mountain State, refused ______ until 1791.
A. to join the Union B. joining the Union C. the joining of the Union D. join the Union
12. It seems that the Earth is the only planet ______.
A. that can support life B. which supports the life
C that can be supported life D. which support life
13. What's the name of the man ______ car you borrowed?
A. who B. which C. that D. whose
14. Are these the keys ______ yesterday?
A. you look for B. you are looking for C. you were looking for D. you had looked for
15. What's the name of the hotel ______?
A. you told me B. you told me about C. you said me about D. you spoke me about
16. Some of the people ______ couldn't come.
A. that I invited to my party B. that I invited my party to
Chuẩn bò kiến thức ôn thi tốt nghiệp THPT và tuyển sinh ĐH, CĐ năm 2008-2009
Nguyen Thanh Tn – GV Tiêng Anh – THPT T_ Ky - T_ Ky - H_i Dư_ng Page 15 of 40
C. to that I invited my party D. which I invited to party
17. Tell me ______ you want, and I'll try to get it for you.
A. that do B. what do C. that if D. what
18. Sheila, ______ , finally arrived.
A. who we've been waiting for B. for whom we'd been waiting
C. whom we'd been waiting D. that we'd been waiting for
19. Fortunately we had a map, ______ we'd have got lost.
A. without which B. with which C. for which D. of which
20. Doctors say that smoking and drinking can both ______ your health.
A. destroy B. improve C. ruin D. damage

21. Air constricted between the vocal chords makes them ______, producing sounds.
A. to vibrate B. vibrating C. vibrate D. the vibration
22. The average spoken sentence in conversational English takes 2.5 seconds ______.
A. for to complete B. completing C. to complete D. by completing
23. In a supermarket, you can use a ______ or pay in cash.
A. credit card B. post card C. charge card D. name card
24. Someone who serves customers at a shop is called a ______.
A. shop master B. shop seller C. shop dealer D. shop keeper
QUESTION 25: The sentences (1-2-3-4-5-6-7) in the following paragraph are in the wrong order. Choose
the correct order to make a good paragraph by circling letter A, B, C or D. Mark your answer on the
answer sheet.
(1) The air is fresh and you have a chance to enjoy nature. (2) Many busy people think it is the most
relaxing kind of holiday. (3) It is much cheaper to hire a caravan or a tent than it is to stay in a motel. (4)
There are several reasons why camping is very popular with a lot of people. (5) Moreover, it is good for us
all to escape from the telephone, the radio, and the television and to forget the problems of the world. (6) For
one thing it is a fairly cheap holiday. (7) In addition it is very pleasant to sleep out in the bush and to have a
picnic meals instead of eating inside.
A. 3-1-2-4-5-6-7 B. 4-6-3-7-1-5-2 C. 2-3-1-4-7-6-5 D. 7-3-2-1-5-6-4
QUESTIONS 26 - 30: Read the following passage carefully and choose the best answer to each question
by circling the corresponding letter A, B, C or D. Mark your choice on the answer sheet.
Tampa, Florida, owes a great deal of its growth and prosperity to a Cuban cigar manufacturer named
Vicente Martinez Ybor. When the Cuban Revolution broke out in 1869, he was forced to flee his country
and moved his business to south Florida. Sixteen years later, serious problems caused him to seek a better
location along the west coast of the state. His original land purchased of sixteen blocks expanded to more
than one hundred acres near Tampa. This newly developed area was called Ybor city in his honor. With the
demand for factory workers for Ybor's business, the surrounding areas expanded and thrived.
26. Where is Ybor city located?
A. south Florida B. Cuba C. west Florida D. in the Florida countryside
27. In what year was Ybor forced to leave south Florida?
A. 1854 B. 1869 C. 1885 D. 1895

28. When did the Cuban Revolution break out?
A. 1869 B. 1896 C. 1886 D.1816
29. Why will people probably continue to remember Ybor's name?
A. He suffered a great deal. B. An area was named in his honor.
C. He was a Cuban revolutionary. D. He was forced to flee his homeland.
30. Why did the surrounding areas expand and thrive?
A. because of the demand for cigars B. because of the demand for factory workers for Ybor's business
C. because of the demand for land D. because of the demand for housing blocks
QUESTIONS 31 - 40: Read the following passage carefully and then choose the best option to fit each
space. Identify your choice by circling letter A, B, C or D on the answer sheet.
"My home is in the air - I do an enormous amount of travelling. It is a fast life and (31) ______ of work,
but I like it and that is the only way (32) ______ me. Every thing is tiring - music, travelling - but what can I
do? I am not (33) ______ to complaining. It is hard to imagine now (34) ______ I will ever be very long in
one place. My home town is on the Caspian Sea. There is sea, wind, sun and (35) ______ many tourists and
hotels. I have my own flat with four or five rooms, but I am seldom there. If I am there for a day or two I prefer
Chuẩn bò kiến thức ôn thi tốt nghiệp THPT và tuyển sinh ĐH, CĐ năm 2008-2009
Nguyen Thanh Tn – GV Tiêng Anh – THPT T_ Ky - T_ Ky - H_i Dư_ng Page 16 of 40
to (36) ______ mother and grandmother. They live in a small house, (37) it is very comfortable and my
mother cooks for me. I like good, simple food. I have no wife, no brothers or sisters and my father (38)
______ seven. He was an engineer and 1 don't (39) ______ well. He liked music very much and wanted me
to (40) ______ an."
31. A. most B. full C. complete D. more
32. A. for B. to C. in D. by
33. A. wanted B. taken C used D. known
34. A. and B. so C. while D. that
35. A. far B. too C. much D. more
36. A. stay B. go C. do D. spend
37. A. but B. since C. even D. which
38. A. killed B. gone C. passed D. died
39. A. know B. remember C. remind D. see

40. A. become B. turn C. develop D. grow
QUESTIONS 41 - 44: Choose one option A, B, C or D corresponding to the sentence which has the same
meaning as the original one and mark your choice on the answer sheet.
41. There will be a shortage of water unless it rains.
A. If it didn't rain, there will be a shortage of water.
B. If it doesn't rain, there would be a shortage of water.
C. If it doesn't rain, there will be a shortage of water.
D. If it rains, there will be a shortage of water.
42. Jane can swim further than I can.
A. I can't swim as far as Jane. B. I can swim as far as Jane.
C. Jane can't swim as far as I can. D. Jane can swim as far as I can.
43. They went out in spite of the heavy rain.
A. Although it rains heavily, they went out. B. Although it rained heavy, they went out.
C. Although it rained heavily, they went out. D. Although it rains heavy, they went out.
44. The flight to Washington lasted more than three hours.
A. It take more than three hours to fly to Washington.
B. It took more than three hours fly to Washington.
C. It took more than three hours to fly to Washington.
D. It took three hours to fly to Washington.
QUESTIONS 45 - 46: The words in italic below are cues. Make sentences by choosing from the
alternatives A, B, C, and D the one best way to make a meaningful sentence. Identify your answer by
circling the corresponding letter A, B, C or D. Mark your choice on the answer sheet.
45. most soaps / detergents / contain phosphates, substance / damaging / environment.
A. Most soaps and detergents contain phosphates, a substance that is damaging to the environment.
B. Most soaps and detergents contain phosphates, a substance is damaging the environment.
C. Most soaps and detergents contain phosphates, substance that is damaging to environment.
D. Most soaps and detergents contain phosphates, a substance that is damaging to environment.
46. we / now / faced / problem / deforestation.
A. We are now faced with problem of deforestation.
B. We are now facing the problem of deforestation.

C. We now faced with problem of deforestation.
D. We now faced with the problem in deforestation.
QUESTIONS 47 - 50: Each of the following sentences has four underlined words or phrases marked A,
B, C, and D. Choose the word or phrase which must be changed in order for the sentence to be correct by
circling letter A, B, C or D and then mark your choice on the answer sheet.
47. Buying clothes are (A) often a very time-consuming (B) practice because those (C) clothes that a person
likes are rarely the ones (D) that fit him or her.
48. Because (A) they had spent too many (B) time considering (C) the new contract, the students lost the
opportunity (D) to lease the apartment.
49. These (A) televisions are all too expensive (B) for we (C) to buy at this time (D), but perhaps we will
return later.
50. After she had bought (A) himself (B) a new automobile, she sold (C) her (D) bicycle.
Chuẩn bò kiến thức ôn thi tốt nghiệp THPT và tuyển sinh ĐH, CĐ năm 2008-2009
Nguyen Thanh Tn – GV Tiêng Anh – THPT T_ Ky - T_ Ky - H_i Dư_ng Page 17 of 40
PRACTICE TEST 7
QUESTIONS 1-2: Choose one word whose underlined part is pronounced differently from the rest in
each group. Identify your answer by circling the corresponding letter A, B, C or D. Mark your choice on
the answer sheet.
1. A. flour B. pour C. hour D. sour
2. A. farming B. harmful C. charity D. garden
QUESTIONS 3-5: Choose one option A, B, C or D corresponding to the word whose main stress is on the
syllable in the position which is different from that of the others. Identify your choice by circling letter A,
B, C or D on the answer sheet.
3. A. finish B. pastime C. summer D. begin
4. A. holiday B. certainty C. industry D. adventure
5. A. increasing B. implying C. interesting D. important
QUESTIONS 6 - 24: These are incomplete sentences. Choose A, B, C or D as your best choice to
complete each sentence and then identify your answer by circling letter A, B, C or D on the answer sheet.
6. I am sorry that I didn't follow your advice, I ______ what you said.
A. should do B. should have done C. might have done D. must do

7. I walked into a wall. I ______ where I was going.
A. should look B. should have been looking
C. must look D. had to look
8. Van Gogh's Sunflowers is ______ $ 39.9 million, three times the previous record.
A. once sold for B. for sale once C. selling for once D. for one sold
9. Some monkeys, ______ , use their tails in a way similar to a hand.
A. like the spider monkey B. spider monkey likes
C. to the spider monkey D. the monkey likes the spider
10. It is gravity ______ objects toward the Earth.
A. pulling B. that pulls C. to pull D. what pulls
11. Using a globe can be ______ it is educational.
A. enjoyable B. to enjoy as C. as enjoyable D. as enjoyable as
12. Most accidents in the home can be prevented by ______ elimination of hazards.
A. that B. that the C. there is a D. the
13. ______ problems in sailing in tropical seas is the coral reefs.
A. One of the biggest B. The biggest one C. Of the biggest one D. There are the biggest
14. ______ their territories but rather than fight, they howl.
A. Wolves protectively jealous B. Jealous of wolves
C. Protection of wolves D. Wolves jealously protect
15. Things are not going so well for her. She has ______ .
A. a few problems B. a little problem C. a few problem D. not problems
16. The growth of two-income families in the United States ______ of people moving to a new social class.
A. has resulted in millions B. results of millions
C. millions of results D. resulting in millions
17. The reason ______ I'm calling was that I didn't know your address.
A. what B. which C. why D. when
18. We'll be late ______ .
A. unless we hurry B. if we aren't hurry C. if we hurry D. unless we don't hurry
19. You can drive my car ______ you drive carefully.
A. unless B. as long as C. because D. although

20. The weather is ______ .
A. depressing B. depressed C. depresses D. depression
21. The book was ______ I couldn't put it down.
A. very good that B. so good than C. so good that D. so good as
22. Birds all over the world ______ in distance up to thousands of miles.
A. migrating B. migrated C. migrate D. are migrated
23. General speaking, every person ______ the potential to be a teacher, to some extent.
A. has B. to have C. having D. have
24. ______ of commodities began in the 1920s at the same time as airmail service.
Chuẩn bò kiến thức ôn thi tốt nghiệp THPT và tuyển sinh ĐH, CĐ năm 2008-2009
Nguyen Thanh Tn – GV Tiêng Anh – THPT T_ Ky - T_ Ky - H_i Dư_ng Page 18 of 40
A. The shipping B. A ship C. The shipped D. To ship
QUESTION 25: The sentences (1-2-3-4-5-6) in the following paragraph are in the wrong order. Choose
the correct order to make a good paragraph by circling letter A, B, C or D. Mark your answer on the
answer sheet.
(1) Moreover, because of school canteens some play grounds are littered with empty cartons and cans.
(2) In the first place, most of the food they sell is "junk food", which is bad for your teeth and not very
healthy. (3) Personally, I think that they should all be closed. (4) This food is not as nutritious as the food
your Mum gives you. (5) School canteens have several disadvantages. (6) As well, students spend far too
much money on stuff they do not need.
A. 5-2-4-1-6-3 B. 4-6-3-1-5-2 C. 2-3-1-4-6-5 D. 3-2-1-5-6-4
QUESTIONS 26 - 30: Read the following passage and complete the sentences that follow by circling the
corresponding letter A, B, C or D. Mark your choice on the answer sheet.
I am often asked whether we will do anything about the taxes we all have to pay. Of course, what
people mean is that they don't want to pay so much. But, I have to ask in return which services they would
like to lose. For we can not expect to have modem hospitals, well- kept roads, good schools or attractive
parks if we are not prepared to pay for these things. Our children do not think that schools and colleges are
only for the sons and daughters of the rich, as once they were. More and more of us are living longer and
longer. We all feel that we should be able to see a doctor when we need to. Old people should not suffer
because we are afraid of the doctor's bill. In our society, we are proud of the fact that no one is asked

whether they can pay the bill before they are allowed into a hospital. But if we want to continue to provide
for all the needs of our society, we must remember that nothing in life is free. My party does not promise to
cut taxes immediately. But we do promise to continue all the services that our taxes pay for, and we hope
that we will manage things so well that after a short time some taxes may be reduced.
26. This is from ____________ .
A. a teacher's diary B. a personal letter C. a political message D. a medical book
27. What is the writer's intention?
A. to explain his ideas B. to describe the past
C. to warn about money problems D. to ask for advice
28. The writer thinks people who complain about taxes should _____________ .
A. remember that services cost money B. realise that services cost nothing
C. pay taxes for welfare services D. pay for their hospital bills
29. The attitude of modern children towards education is that _____________ .
A. education is only for children of the rich
B. education is only for children of the disabled
C. education is for all children
D. education is only for children of the poor
30. One of these people shares the same ideas as the writer. Whose idea is it?
A. I just don't know where I'm going to find the money I need. There's so much to pay for.
B. The trouble with people nowadays is that they all want something for nothing. They think they
should be able to get whatever they want. But they don't realise that some one is going to have to
pay for it in the end.
C. I know I ought to go and see the doctor, but I don't think I can afford it. I know he will tell me to
buy some medicine, but I just haven't got the money.
D. What the government ought to do is give people control of their own money, if we didn't have to
pay taxes we'd be able to afford to pay all our own bills.
QUESTIONS 31 - 40: Read the following passage carefully and then choose the best option to fit each
space. Identify your choice by circling letter A, B, C or D on the answer sheet.
Scientists used to believe that our 24-hour cycle of sleeping and waking was governed entirely by
external factors. The most notable of these, they thought, were the rising and (31) ______ of the sun. But

they have now (32) ______ that there is a daily rhythm to a (33) ______ range of biological functions -
including temperature, digestion and mental (34) – s which are regulated internally by a special timekeeping
mechanism within the brain.
The main function of this "body clock" is to anticipate and (35) ______ for external changes so that, for
example, body temperature starts to rise (36) ______ dawn, gearing us up for the day, and begins to (37) in
the early evening, winding us down for sleep.
Chuẩn bò kiến thức ôn thi tốt nghiệp THPT và tuyển sinh ĐH, CĐ năm 2008-2009
Nguyen Thanh Tn – GV Tiêng Anh – THPT T_ Ky - T_ Ky - H_i Dư_ng Page 19 of 40
Some people's body clocks (38) poorer time than others, which can greatly disturb their lives and even
(39) ______ their health. Insomnia, depression, fatigue, poor work performance and even accidents can all
be (40) - or aggravated by inaccurate body clocks.
31. A. descending B. diving C. dipping D. setting
32. A. established B. fixed C. settled D. assured
33. A. wide B. various C. far D. grand
34. A. operation B. activity C. process D. occupation
35. A. dispose B. scheme C. steady D. prepare
36. A. beside B. approximately C. around D. nearly
37. A. fall B. reduce C. lessen D. subtract
38. A. keep B. hold C. support D. preserve
39. A. risk B. spoil C. injure D. threaten
40. A. put B. formed C. caused D. made
QUESTIONS 41 - 42: Choose one option A, B, C or D corresponding to the sentence which has the same
meaning as the original one and mark your choice on the answer sheet.
.41. They've translated Nguyen Du's novel "Kieu" into several languages.
A. Nguyen Due’s novel "Kieu" has been translated into several languages.
B. Nguyen Du's novel "Kieu" has translated into several languages.
C. Nguyen Du's novel "Kieu" had translated into several languages.
D. Nguyen Du's novel "Kieu" was been translated into several languages.
42. They sold computers commercially for the first time in the 1950s.
A. Computers are sold commercially for first time in the 1950s.

B. Computers were sold commercially for the first time in the 1950s.
C. Computers have been sold commercially the first time in the 1950s.
D. Computers was sold commercially for the first time in the 1950s.
QUESTIONS 43 - 45: After each pair of simple sentences from 43 to 45, there are four options which are
their combinations. Choose the best answer by circling the corresponding letter A, B, C or D. Mark your
choice on the answer sheet.
43. I received a letter this morning. It really upset me.
A. I received a letter this morning really upset me.
B. I received a letter this morning which really upset me.
C. I received a letter this morning in which really upset me.
D. I received a letter this morning of which really upset me.
44. It was a great summer. I'll never forget that summer.
A. It was a great summer that I'll never forget.
B. It was a great summer I'll never forget that.
C. It was a great summer of which I'll never forget.
D. It was a great summer of that I won't never forget.
45. Hong is going to tell us about a terrible storm. She is from Vinh City.
A. Hong, who is from Vinh City, is going to tell us about a terrible storm.
B. Hong, who is from Vinh City, is going to tell us terrible storm.
C. Hong, that is from Vinh City, is going to tell us about a terrible storm.
D. Hong, that is from Vinh City, is going to tell about us a terrible storm.
QUESTIONS 46 - 48: The words in italic below are cues. Make sentences by choosing from the
alternatives A, B, C, and D the one best way to make a meaningful sentence. Identify your answer by
circling the corresponding letter A, B, C or D. Mark your choice on the answer sheet.
46. surfing / net / I / enjoy / but / have / I / don't / time / it.
A. I enjoy surfing net but I don't have many time for it.
B. I enjoy surfing the net but I don't have few time for it.
C. I enjoy surfing on the net but I don't have much time for it.
D. I enjoy surfing on the net but I don't have a few time for it.
47. What / you / like / do / your / free time?

A. What do you like in your free time? B. What do you like doing in your free time?
C. What you like doing in your free time? D. What do you like doing your free time?
48. What /favourite / TV / programs?
Chuẩn bò kiến thức ôn thi tốt nghiệp THPT và tuyển sinh ĐH, CĐ năm 2008-2009
Nguyen Thanh Tn – GV Tiêng Anh – THPT T_ Ky - T_ Ky - H_i Dư_ng Page 20 of 40
A. What are your favourite TV programs? B. What is your favourite TV programs?
C. Which is your favourite TV programs? D. What's your favourite TV programs?
QUESTIONS 49 - 50: Each of the following sentences has four underlined words or phrases marked A,
B, C, and D. Choose the word or phrase which must be changed in order for the sentence to be correct by
circling letter A, B, C or D and then mark your choice on the answer sheet.
49. Cycling (A) helmets (B) are not a legal (C) require (D) in Britain.
50. I suggested that it ought to have an entertaining (A) page with the lists (B) of what's on at local (C)
cinemas (D).
PRACTICE TEST 8
QUESTIONS 1-2: Choose one word whose underlined part is pronounced differently from the rest in
each group. Identify your answer by circling the corresponding letter A, B, C or D. Mark your choice on
the answer sheet.
1. A. gym B. game C. gone D. gather
2. A. garment B. hardship C. carry D. garlic
QUESTIONS 3-5: Choose one option A, B, C or D corresponding to the word whose main stress is on the
syllable in the position which is different from that of the others. Identify your choice by circling letter A,
B, C or D on the answer sheet
3. A. popular B. dangerous C. magazine D. applicant
4. A. battle B. career C. engine D. rabies
5. A. people B. possess C. purpose D. pepper
QUESTIONS 6 - 24: These are incomplete sentences. Choose A, B, C or D as your best choice to
complete each sentence and then identify your answer by circling letter A, B, C or D on the answer sheet.
6. Ann is going to write ______ of articles for her local newspaper.
A. a series B. the series C. a serie D. the serie
7. She climbed out of the swimming pool and ______ with a towel.

A. drying herself B. dried herself C. dries herself D. having dried herself
8. The movie itself wasn't very good, ______ I liked the music.
A. and B. or C. but D. so
9. Who was Tom with when you saw him? "Nobody." He was ______.
A. in his own B. by his own C. on his own D. with his own
10. Mary doesn't buy many clothes. She usually ______.
A. makes by herself B. makes her clothes C. makes her own clothes D. makes clothes her own
11. Are you ______ in soccer?
A. interesting B. interests C. interested D. interest
12. He works very hard. It's not ______ that he's always tired.
A. surprised B. surprising C. surprise D. surprises
13. He's one of the most ______ people I've ever met.
A. bored B. bore C. boring D. boredom
14. About twenty miles from Boston, ______ a little town, named Concord, that has a rich history.
A. has B. there is C. there are D. where is
15. There was a ______ table in the kitchen.
A. beautiful large round wooden B. large beautiful round wooden
C. beautiful round large wooden D. wooden round large beautiful
16. He's a man with a ______.
A. thin long face B. face long and thin C. long thin face D. thinly long face
17. I kept them in the ______ .
A. metal black small box B. small black metal box C. black metal small box D. small metal black box
18. They spoke ______ for us to understand.
A. slow enough B. so slowly C. slowly so D. slowly enough
19. It's too noisy here. Can we go somewhere ______.
A. quietly B. more quietly C. quieter D. quite
20. It was ______ we spent the whole day at the beach.
A. such a nice weather that B. such nice weather that
C. such nice a weather than D. such a nice weather so
Chuẩn bò kiến thức ôn thi tốt nghiệp THPT và tuyển sinh ĐH, CĐ năm 2008-2009

Nguyen Thanh Tn – GV Tiêng Anh – THPT T_ Ky - T_ Ky - H_i Dư_ng Page 21 of 40
21. It is only recently that ballets have been based on themes ______ American life.
A. reflecting B. reflects C. is reflecting D. reflected
22. Poison oak generates irritating poisons ______ even if people merely brush against the plants.
A. they can affect people B. that can affect people
C. what can affect people D. which do they affect
23. ______ is no way to tell the exact number of heroin addicts in Vietnam.
A. It B. There C. What D. Each
24. In order for people to work together effectively, they need ______ each other's needs.
A. to be sensitive to B. is sensitive for C. sensitivity D. sensitive
QUESTIONS 25: The sentences (1-2-3-4-5-6-7) in the following paragraph are in the wrong order.
Choose the correct order to make a good paragraph by circling letter A, B, C or D. Mark your answer on
the answer sheet.
(1) They do not want to wear the same dark blue or yellow and brown that everyone else is wearing. (2)
Schools uniforms should be optional for these reasons. (3) Another problem is colour. (4) For one thing, they
are very expensive nowadays especially for teenagers. (5) Boys and girls nowadays like to express
themselves by wearing different colored clothes that are available in the shops. (6) For another thing,
although the styles may suit one student it does not often suit another. (7) So I think that we should be
allowed to choose for ourselves what we wear for most of the day.
A. 5-4-1-6-3-2-7 B. 4-6-3-1-5-2-7 C. 2-4-6-3-5-1-7 D. 3-2-1-5-6-4-7
QUESTIONS 26 - 30: Read the following passage and then choose the best answer to the questions by
circling the corresponding letter (A, B, C or D). Mark your choice on the answer sheet.
Although the composition and role of the board of directors of a company will vary from one
organization to the next, a few generations may be made. As regards the composition of the board,
customarily some directors are prominent men and women selected to give prestige to the group. Others are
usually chosen from among retired executives of the organization for their specialized knowledge of the
company.
It is generally true that, as long as the top management maintains the confidence of the board of
directors, the directors will not actively intervene to dictate specific policies. This is the same administrative
procedure usually followed by the board of trustees of a college or university, and is similar in many

respects to the parliamentary system of ministerial responsibility practised in Great Britain.
26. The title that best expresses the ideas in this passage is ____________ .
A. The Board of Directors B. The Board of Trustees
C. The Parliamentary System D. Management
27. Who generally formulates policies for a company?
A. Top management B. A dictator C. The board of directors D. Retired executives
28. What does the author mean by the statement "It is generally true that, as long as the top management
maintains the confidence of the board of directors, the directors will not actively intervene to dictate
specific policies."?
A. Policies are dictated by the board with the approval of top management.
B. The board will assume control only if they loose confidence in management.
C. A vote of confidence by management authorizes the board to make policies.
D. When management loses confidence in the board, they will assume control.
29. The author mentions all the following as having similar administrative procedures EXCEPT
A. the board of directors of a company B. the board of trustees of a college
C. the members of Congress in the United States D. the members of Parliament in Great Britain
30. Who would not be a likely candidate to be chosen as a member of the board of directors of City Bank?
A. a retired president of City Bank B. a respected lawyer
C. a senator D. a City Bank employee
QUESTIONS 31 - 40: Read the following passage carefully and then choose the best option to fit each
space. Identify your choice by circling letter A, B, C or D on the answer sheet.
Most British people go abroad on holiday, to visit family or on short business trips. People are (31)
______ to find out how to get urgent treatment before leaving the U.K. They have to (32) ______ a form
which explains what they (33) ______ do if they fall ill or (34) ______ an accident, and what
arrangements exist in (35) ______ country for medical treatment. The regulations are fairly simple but (36)
______ people do not have this information, they may (37) ______ that private medical care is extremely
Chuẩn bò kiến thức ôn thi tốt nghiệp THPT và tuyển sinh ĐH, CĐ năm 2008-2009
Nguyen Thanh Tn – GV Tiêng Anh – THPT T_ Ky - T_ Ky - H_i Dư_ng Page 22 of 40
expensive. It is not unusual for people to discover that they do not have (38) ______ money with them to
(39) ______ the total costs and (40) ______ such circumstances an already difficult situation becomes even

more complicated.
31. A. advised B. suggested C. said D. spoken
32. A. put B. bring C. fill D. get
33. A. ought B. will C. should D. may
34. A. have B. get C. happen D. take
35. A. their B. each C. one D. this
36. A. because B. whether C. as D. if
37. A. look B. want C. find D. know
38. A. enough B. little C. few D. full
39. A. pay B. give C. spend D. have
40. A on B. in C. at D. up
QUESTIONS 41 - 45: Choose one option A, B, C or D corresponding to the sentence which has the same
meaning as the original one and mark your choice on the answer sheet.
41 .It's a pity. 1 can't play chess.
A. I wish I can play chess. B. I wish I could played chess.
C. I wish I could play chess. D. I wish I play chess.
42. "Where does Mai live? " he asked.
A. He asked me where Mai lives. B. He asked me where does Mai live.
C. He asked me where Mai lived. D. He asked me where did Mai live.
43. They are going to celebrate their 25lh wedding anniversary in Hue.
A. Their 25lh wedding anniversary are going to celebrate in Hue.
B. Their 25th wedding anniversary is going to be celebrated in Hue.
C. Their 25lh wedding anniversary are going to be celebrating in Hue.
D. Their 25th wedding anniversary is going to be celebrate in Hue.
44. Lan doesn’t have enough time to do it well.
A. If Lan had more time, she would do it well. B. If Lan had more time, she will do it well.
C. If Lan had more time, she would have done it well D. If Lan had more time, she did it well.
45. I want to buy him a T-shirt on his birthday but I haven 't got any money.
A. I'd buy him a T-shirt on his birthday if t had some money.
B. I'd buy him a T-shirt on his birthday if I had few money.

C. I'd buy him a T-shirt on his birthday if I have some money.
D. I'd buy him a T-shirt on his birthday if I have few money.
QUESTIONS 46 - 50: Each of the following sentences has four underlined words or phrases marked A,
B, C, and D. Choose the word or phrase which must be changed in order for the sentence to be correct by
circling letter A, B, C or D and then mark your choice on the answer sheet.
46. Some of the plants (A) in this store require very little care (B), but this one needs much more sunlight
(C) than the others ones (D) .
47. Daniel said that if he had (A) to do another (B) homework tonight, he would not be able to (C) attend
(D) the concert.
48. John decided to buy (A) in the morning a new car (B), but in the afternoon (C) he changed his mind (D).
49. It has always been (A) her dreamt (B) to write (C) a successful (D) novel.
50. Water (A) covers (B) two- three (C) the area of (D) the world.
PRACTICE TEST 9
QUESTIONS 1-2: Choose one word whose underlined part is pronounced differently from the rest in
each group. Identify your answer by circling the corresponding letter A, B, C or D. Mark your choice on
the answer sheet.
1 A. cheer B. chemist C. check D. chat
2. A. fly B. sky C. spy D. puppy
QUESTIONS 3-5: Choose one option A, B, C or D corresponding to the word whose main stress is on the
syllable in the position which is different from that of the others. Identify your choice by circling letter A,
B, C or D on the answer sheet.
3. A. important B. comfortable C. surprising D. extinction
Chuẩn bò kiến thức ôn thi tốt nghiệp THPT và tuyển sinh ĐH, CĐ năm 2008-2009
Nguyen Thanh Tn – GV Tiêng Anh – THPT T_ Ky - T_ Ky - H_i Dư_ng Page 23 of 40
4. A. carry B. invent C. appoint D. become
5. A. sentence B. season C. success D. saucer
QUESTIONS 6 - 24: These are incomplete sentences. Choose A, B, C or D as your best choice to
complete each sentence and then identify your answer by circling letter A, B, C or D on the answer sheet.
6. I'd like to have a(n) ______ car. Mine keeps breaking down.
A. reliably B. unreliable C. more reliably D. more reliable

7. I ran ______ Dan did.
A. faster than B. fastly than C. more fastly than D. more faster than
8. Her illness was ______ we thought at first.
A. much serious than B. seriouser than C. much more serious than D. much seriously than
9. Do you feel ______ today?
A. a lot of better B. some better C. any better D. the better
10. Peter often drives a ______.
A. little red old car B. red little old car C. old little red car D. little old red car
11. Linda likes wearing ______ clothes.
A. colour B. colourful C. colourfully D. colourness
12. They asked me a lot of questions, ______ I couldn't answer.
A. much of which B. most of which C. both of them D. neither of which
13. The boy ______ in the accident was taken to the hospital.
A. injures B. injured C. injuring D. injure
14. The machine ______ has now been repaired.
A. that broken down B. that broke down C. which is broken down D. which broke
15. The man ______ on the plane talked all the time.
A. who I am sitting next to B. that I was sitting next to him
C. that I was sitting next to D. who I was sitting next him
16. He was ______ the job.
A. experienced enough to do B. enough experienced to do
C. experienced enough doing D. experienced to do enough
17. I have ______ , but I don't have time.
A. a vacation enough money for B. money enough for a vacation
C. enough money for a vacation D. enough money for vacation
18. They are not interested ______.
A. in the internet B. exploring the internet
C. in play games in the internet D. surfing on the net
19. We don't like ______.
A. play basket ball B. playing basket ball C. played the basket ball D. plays basket ball

20. They are not coming, ______ ?
A. are they B. aren't they C. they are D. don't they
21. Earnest Hemingway is ______ of modern fiction.
A. one of the molders B. who is one of the molders
C. the molders one D. the molders who is the one
22. ______ occasions for congratulations.
A. Birthdays that usually considered B. Birthdays are usually considered
C. Usually considering birthday D. That considered birthdays usually
23. It is good form to use the name of the person ______.
A. who are greeting B. which you are greeting
C. you are greeting D. greeting for you
24. It is possible ______ may assist some trees in saving water in the winter.
A. the leaves are lost B. that the loss of leaves
C. when leaves have lost D. to lose leaves
QUESTIONS 25: The sentences (1-2-3-4-5) in the following paragraph are in the wrong order. Choose
the correct order to make a good paragraph by circling letter A, B, C or D. Mark your answer on the
answer sheet.
(1) The family of the patient is responsible for these expenses. (2) Consequently, they would be a terrible
financial burden for them for a long time. (3) The costs of medicines and medical tests are also high. (4) One
Chuẩn bò kiến thức ôn thi tốt nghiệp THPT và tuyển sinh ĐH, CĐ năm 2008-2009
Nguyen Thanh Tn – GV Tiêng Anh – THPT T_ Ky - T_ Ky - H_i Dư_ng Page 24 of 40
of the reasons for terminally ill patients to be allowed to die is that medical costs are very high. (5) The cost
of a hospital room can be as much as five hundred dollars per day and even more.
A. 5-4-1-3-2 B. 4-5-3-1-2 C. 2-4-5-3-1 D. 3-2-1-5-4
QUESTIONS 26 - 30: Read the following passage and then choose the best answer to the questions by
circling the corresponding letter (A, B, C or D). Mark your choice on the answer sheet.
The Hopi are part of the Pueblo Indian culture. Today they live mostly in northeastern Arizona, at the
edge of the Painted Desert. Something that sets the Hopi off from others cultures is that it is in some senses a
maternal rather a paternal culture.
The Hopi are divided into clans, or families, along maternal lines, and, as a result, a child becomes a

member of the mother's clan rather than the father's. In addition, ownership of property, such as land and
houses, passes from mother to daughter instead of from father to son, as it does in other Native American
cultures. However, women do not have all the power in this culture. Societal authority still rests in the hands
of men, but that authority does pass to men from their mothers.
26. The main idea of the passage is that _____________.
A. the Hopi are one type of Pueblo Indian B. the Hopi have a maternal culture
C. no Indian cultures are paternal D. today the Hopi live in northeastern Arizona
27. The passage states that, __________ .
A. the Hopi do not own property
B. the Hopi's property passes from father to son
C. the Hopi live inside the Painted Desert
D. the Hopi may be found on the borders of the Painted Desert
28. In line 3, something that is "maternal" is related to________________ .
A. the culture B. the mother C. the Hopi D. the clan
29. A clan in line 5 is a ____________ .
A. mother B. father C. family D. child
30. Which of the following is true about Hopi passage'.
A. Hopi do not own property
B. Hopi's property passes from father to son
C. Property ownership in the Hopi culture is similar to property ownership in most other cultures
D. A Hopi daughter will probably inherit property from her mother
QUESTIONS 31 - 40: Read the following passage carefully and then choose the best option to fit each
space. Identify your choice by circling letter A, B, C or D on the answer sheet.
C & A is probably one of the largest family fashion stores in the world, and their shops are a familiar
sight in (31) ______ High Streets in Britain. But many British people would be surprised to learn (32)
______ this fashion company has Dutch origins. Founded in the small market town of Sneek in 1841, the
first C & A shop (33) ______ opened by two brothers called Clemens and August Brenninkmeyer. The
initials of their first names formed the name of the shop. As trade grew, more (34) ______ were opened
across Europe and in 1922 C & A came to Britain. (35) ______ success was immediate and the store was a
major influence in bringing down the (36) ______ of women's clothes. It was the two brothers (37) ______

started the 5- day working week, at a time when shopkeepers in Britain expected their (38) ______ to work a
6- day week. Today C & A employs many thousands of people. All the stores are attractively (39) ______
with good use of space, lighting and plenty of individual changing rooms where (40) ______ can try things
on in comfort before they buy.
31. A. every B. most C. more D. each
32. A. that B. if C. and D. because
33. A. were B. have C. had D. was
34. A. stores B. places C. parts D. houses
35. A. They B. This C. That D. Their
36. A. value B. figures C. cost D. money
37. A. which B. who C. what D. whom
38. A. staff B. groups C. officers D. employers
39. A. drawn B. designed C. set D. put
40. A. customers B. patients C. one D. persons
QUESTIONS 41 - 43: After each pair of simple sentences from 41 to 43, there are four options which are
their combinations. Choose the best answer by circling the corresponding letter A, B, C or D. Mark your
Chuẩn bò kiến thức ôn thi tốt nghiệp THPT và tuyển sinh ĐH, CĐ năm 2008-2009
Nguyen Thanh Tn – GV Tiêng Anh – THPT T_ Ky - T_ Ky - H_i Dư_ng Page 25 of 40
choice on the answer sheet.
41. “Tsunami” is a Japanese word. This word means "harbor wave".
A. "Tsunami" is a Japanese word which it means "harbor wave".
B. "Tsunami" is a Japanese word it means "harbor wave".
C. "Tsunami" is a Japanese word which means "harbor wave".
D. "Tsunami" means "harbor wave" is a Japanese word.
42. Sue lives in the house. The house is opposite my house.
A. Sue lives in the house whose is opposite my house.
B. Sue lives in the house which opposite my house.
C. Sue lives in the house which is opposite her house.
D. Sue lives in the house which is opposite my house.
43. The plants may develop differently. The plants live on that island.

A. The plants live on that island may develop differently.
B. The plants that live on that island may develop differently.
C. The plants which lives on that island may develop differently.
D. The plants that live on that island may develop different.
QUESTIONS 44 - 48: The words in italic below are cues to make sentences. Choose the correct option by
circling the corresponding letter A, B, C or D. Mark your choice on the answer sheet.
44. air / be / also / the / pollute.
A. The air was also polluted. B. The air is also polluted.
C. The air is also pollute. D. The air is also pollution.
45. Human / action / can / lead / greenhouse / effect / the.
A. Human action lead to the greenhouse effect.
B. Human action can lead the greenhouse effect.
C. Human action can lead to greenhouse effect.
D. Human action can lead to the greenhouse effect.
46. Ha / speak / English / work / interpreter / well.
A. If Ha spoke English well, she would work as an interpreter.
B. If Ha spoke English well, she would work an interpreter.
C. If Ha spoke English well, she works as an interpreter.
D. If Ha spoke English well, she will work as an interpreter.
47. Bill / win / lottery / travel / the world / if.
A. Bill would travel around the world if he wins the lottery.
B. Bill would travel around the world if he won the lottery.
C. Bill would travel the world if he won the lottery.
D. Bill will travel around the world if he won the lottery.
48. What / you / do / if / you / meet / alien / outer space?
A. What do you do if you met an alien from outer space?
B. What would you do if you met alien from outer space?
C. What would you do if you met an alien from outer space?
D. What would you do if you meet an alien from outer space?
QUESTIONS 49 - 50: Each of the following sentences has four underlined words or phrases marked A,

B, C, and D. Choose the word or phrase which must be changed in order for the sentence to be correct by
circling letter A, B, C or D and then mark your choice on the answer sheet.
49. I was hoping that you might be able to give (A) me (B) some (C) advise about what I should do (D).
50. Do you think that climate (A) effects (B) people's (C) personalities (D)?

THE END

PRACTICE TEST 10

QUESTIONS 1-5: Choose one option A, B, C or D corresponding to the word whose main stress is on the
syllable in the position which is different from that of the others. Identify your choice by circling letter A,
B, C or D on the answer sheet.
1. A. Canadian B. vegetarian C. pedestrian D. incredible
2. A. edible B. classify C. terrify D. solidify
3. A. associate B. operate C. integrate D. recognize
4. A. ambitious B. dangerous C. mysterious D. expensive
5. A. secretary B. necessary C. classify D. vocabulary
QUESTIONS 6 - 35: These are incomplete sentences. Choose A, B, C or D as your best choice to
complete each sentence and then identify your answer by circling letter A, B, C or D on the answer sheet.
6. The lower ______ is in a room, the more slowly our eyes focus.
A. the level of lighting B. light level C. leveling of light D. lighting is level
7. The wind-rippled sand at California's Kelso Dunes resembles ______.
A. to be an ocean floor B. as an ocean floor C. an ocean floor D. being an ocean floor
8. Living organisms contain more water ______ substance.
A. than do any other B. does than any other C. other than do they any D. than any other
9. Columbia University's School of Public Health is ______ a handful of schools of public health in the
United States.
A. one that B. the one C. one of D. one
10. More than anything, what saved Jamestown was the highly successful cultivation of tobacco, ______
Indian assistance with farming undoubtedly played a major part as well.

A. although B. in spite of C. nor D. neither
11. According to legend, coffee beans ______ in the town of Kaffa.
A. first discovered B. were first discovered C. discovered first D. have first discovered
12. Martin Luther King Jr, after his father's assassination, ______ the mission of his father to fight for the
rights of blacks in America and achieved some notable success in the 1970s.
A. continued B. continuing C. and continues D. is continuing
13. In the symphony orchestra, bass drums are not ______ kettle drums.
A. as prevalent B. that prevalent C. so prevalent as D. prevalent than
14. Narcissus bulbs ______ at least three inches apart and covered with about four inches of well drained
soil.
A. should be planted B. to plant C. must planting D. should plant
15. Total color blindness, ______ , is the result of a defect in the retina.
A. a rare condition that B. a rare condition C. that a rare condition D. is a rare condition
16. In the fourteenth century, ______ that glass coated with silver nitrate would turn yellow when fired in
an oven.
A. the discovery B. it was discovered C. with the discovery D. if it was discovered
17. ______ electric eels send a series of blips of electricity into the water around them and they can detect
the pattern of electricity of the water changes.
A. All of B. Every C. All D. Of each
18. ______ an ownership of nearly 60 to 62 per cent of corporate equities, this mean 1-2 per cent of
American society possesses about 43 per cent of the total wealth of the nation.
A. The range of B. Ranging C. Ranging from D. The range
19. Earthquakes can damage a tree ______ violently, and it can take several years for the tree to heal.
A. to cause shaking B. when shaking it causes
C. by causing it to shake D. to cause to shake it
20. ______ classified as a carnivore, the North American Grizzly bear eats berries and even grass.
A. Just as B. Because of C. Although D. Either
21. The sea horse uses ______ to cling to the seaweed and other plants.
A. it has a tail B. a tail with which it C. its tail D. as its tail
22. The human skin forms ______ against the action of physical, chemical, and bacterial agents on the

deeper tissues.
A. protective a barrier is B. a protective barrier
C. a barrier protect D. a barrier protects
23. The swallows of Capistrano are famous ______ to the same nests in California each spring.
A. to return B. who returned C. they returned D. for returning
24. The invention in the 1920s of the Corning ribbon ______ important, because from that time on the price
of a light bulb plunged.
A. machine was B. while a machine was C. had a machine D. for the machine
25. In 1950, aspirin earned a place in the Guinness Book of World Records ______.
A. in the world's best selling painkiller B. in the world as a best selling painkiller
C. as the world's best-selling painkiller D. of the world's best selling painkiller
26. Perhaps we should think in terms of raising interest rates ______ them, in consideration of the new
reports about inflation reported last June.
A. then reducing B. and reduce C. although reduce D. rather than reducing
27. ______ bacteria in foods are killed, as they are during baking or stewing, decay is slowed down.
A. What B. The C. If D. So
28. The Pulitzer Prize has been ______ in American literature for more than seventy years.
A. the award most prestigious that B. the most prestigious award
C a prestigious award that most D. most prestigious award
29. Spain and Portugal's competition for economic domination in the Americas ______ after
Columbus' return to Europe, when they both realized that the Newland could increase their gold
reserves.
A. began B. begin C. begun D. was beginning
30. The lenses in an optical microscope bend the light passing through a specimen to form an image of that
specimen that is much larger ______ actually viewed.
A. than it B. than the one C. one than D. than one which
31. In this pure state antimony has no important uses, but ______ with other substances, it is an extremely
useful metal.
A. when combined physically or chemically B. combined when physically or chemically
C. the physical and chemical combination D. it is combined physically and chemically

32. In hot, dry regions, the Sun's heat causes the outer layer of rocks ______, a process called exfoliation.
A. are expanded and peeled away B. to expand and peel away
C. expands and peels away D. they expand and peel away
33. Industrialization has been responsible for ______ most radical of the environmental changes caused by
humans.
A. a B. the C. some of which D. which are the
34. For the first time in 70 years, ______ aspirin's potential beyond reducing pain, fever and inflammation.
A. researchers began to understand B. researchers begin understanding
C. researcher's understanding of D. researchers began their understanding
35. The hourglass, an instrument used ______ , usually consists of two bulbs united by a narrow neck.
A. measures time B. time is measured C for measuring time D. the time for measuring
QUESTIONS 36 - 45: Read the following passage carefully and then choose the best option to fit each
space. Identify your choice by circling letter A, B, C or D on the answer sheet.
The 18th century battlefield was, compared with that of the 20th century, an intimate theatre, especially
intimate in the (36) ______ of the Revolution, which was usually small even by the standards of the day.
Soldiers had to come to close quarters to kill; this fact reduced the mystery of the battle, though perhaps not
its terrors. But at least the battle field lost some of its impersonality. In fact, in (37) ______ to (with) the 20th
century combat, in which the enemy usually remains unseen and the source of incoming fire unknown, in the
18th century battles the enemy could be seen and sometimes even (38) ______. (39) ______ one's enemy
may have arose a singular (40) ______ of feeling uncommon in modern battles. Before the (41)
______ occurred tension and anxiety (42) ______ up as the troops marched from their column into a line of
attack. The (43) ______ of their movements was well understood by themselves and their enemies, who
must have watched with feeling of dread and fascination. When the order came sending them (44) ______,
rages, even madness replaced the attacker's (45) ______, while terror and desperation filled those who
received the charge.
36. A. success B. engagement C. attachment D. independence
37. A. connection B. contrast C. contract D. link
38. A. stroked B. touched C. seen D. confronted
39. A. Seeing B. Dealing C. Fighting D. Kilting
40. A. despair B. sadness C. intensity D. stress

41. A. assault B. confrontation C. obedience D. order
42. A. stored B. kept C. set D. built
43. A. notion B. purpose C. reason D. trace
44. A. upward B. outward C. forward D. downward
45. A. anxiety B. sympathy C. passions D. nerves
QUESTIONS 46 - 50: Read the following passage carefully and complete the sentences that follow by
circling letter A, B, C or D as the correct answers and then mark your choice on the answer sheet.
Most fairy tales in the world begin with "Once upon a time" and end with "They lived happily ever
after," so we will begin in the same way.
Once upon a time, there was a girl called Cinderella who did all the work in the kitchen while her lazy
sister did nothing.
One night, her sister went to a ball at the palace. Cinderella was left home, very sad. After a time, her
fairy godmother appeared and told Cinderella that she could go to the ball - but to return home by midnight.
So she went to the ball in a beautiful dress in a wonderful coach. She danced with the prince but at
midnight she ran back home, leaving one of her shoes on her dancing floor. The prince wanted to see her
again and went to every house in the capital until he found that the shoe was the right size for Cinderella.
She and the prince were married and lived happily ever after.
46. Books with fairy tales are found in ____________ .
A. our country only B. few countries C. all countries except ours D. many countries
47. According to the passage, fairy tales in different languages usually begin and end ______ .
A. in the same way B. in different ways C. in many ways D. in various ways
48. The word "ball" in the first sentence of paragraph 3 means ______________.
A. a sport equipment B. a dancing hall C. a balloon D. sphere
49. Cinderella was very sad because _____________ .
A. her sister did nothing B. her sister went to a ball and left her at home
C. she did all the work in the kitchen D. her sister was invited to a ball
50. At the end of the story, ____________.
A. Cinderella could go to the ball and its meaning is clear that she and the prince were married
B. Cinderella's godmother came to comfort her C. one of Cinderella's sisters was married to the prince
D. the prince invited Cinderella to the ball

QUESTIONS 51 - 55: Read the following passage carefully and complete the sentences that follow by
circling letter A, B, C or D as the correct answers and then mark your choice on the answer sheet.
In many modem countries, people think of a family as a mother, a father and their children. But this is
not the only kind of the family group. In some parts of the world, a family group has many other members.
This kind of large family is called an "extended family" or a "joint family".
The joint family includes all living relatives on either the mother's or the father's side of the family. It is
made up of grandparents, parents, brothers, sisters, uncles, aunts and cousins. They live together in a large
house or in huts built close together.
Early people probably lived in joint families. They had to be part of a large group in order to survive.
The members of the group help each other hunt.
They work together to protect themselves from dangerous animals and other enemies.
In China, people lived in joint families. When a son married, he and his wife lived at his parents' home.
Unmarried daughters remained at home until they married. Chinese children felt very loyal to their parents.
Younger members of the joint family always took care of the old ones.
In India and Africa, some people still live in joint families. The members of a joint family share their
earnings and property. If one member of the group becomes ill or has bad luck, the others help the person.
As in the past, the members of the joint family offer each other help and protection.
51. The word in paragraph 2 that means "to be made up of is _____________ .
A. include B. relatives C. live D. hut

×